Những câu hỏi liên quan
liên hoàng
Xem chi tiết
Lê Minh Đức
Xem chi tiết
Long Vũ
20 tháng 3 2016 lúc 19:48

\(\frac{1}{1+x}+\frac{1}{1+y}+\frac{1}{1+z}\ge2\Leftrightarrow\frac{1+1+1}{\left(1+1+1\right)xyz}=\frac{3}{3}.xyz=1xyz\)

mà P\(\ge2\)(vô lí)

=> MaxP=xyz=1

Bình luận (0)
Vô Danh
20 tháng 3 2016 lúc 19:55

lời giải sai bét!!

Bình luận (0)
nguyễn thị ngọc minh
20 tháng 3 2016 lúc 20:10

BUI LONG VU suy kieu j ra zay

Bình luận (0)
Ác Quỷ Bóng Đêm
Xem chi tiết
Hoàng Lê Bảo Ngọc
2 tháng 9 2016 lúc 8:40

Xét giả thiết : \(\frac{1}{1+x}+\frac{1}{1+y}+\frac{1}{1+z}\ge2\Leftrightarrow\frac{1}{1+x}\ge\left(1-\frac{1}{1+y}\right)+\left(1-\frac{1}{1+z}\right)\)

\(\Leftrightarrow\frac{1}{1+x}\ge\frac{y}{1+y}+\frac{z}{1+z}\ge2\sqrt{\frac{yz}{\left(1+y\right)\left(1+z\right)}}\)

Tương tự : \(\frac{1}{1+y}\ge2\sqrt{\frac{xz}{\left(1+x\right)\left(1+z\right)}}\) ; \(\frac{1}{1+z}\ge2\sqrt{\frac{xy}{\left(1+x\right)\left(1+y\right)}}\)

Nhân các bđt trên theo vế : \(\frac{1}{\left(1+x\right)\left(1+y\right)\left(1+z\right)}\ge\frac{8xyz}{\left(1+x\right)\left(1+y\right)\left(1+z\right)}\)

\(\Rightarrow1\ge8xyz\Rightarrow xyz\le\frac{1}{8}\)

Dấu "=" xảy ra khi \(\begin{cases}\frac{1}{1+x}+\frac{1}{1+y}+\frac{1}{1+z}=2\\\frac{1}{1+x}=\frac{1}{1+y}=\frac{1}{1+z}\end{cases}\) \(\Leftrightarrow x=y=z=\frac{1}{2}\)

Vậy max (xyz) = 1/8 <=> x = y = z = 1/2

Bình luận (0)
syl tráo nọy lguơì
Xem chi tiết
Incursion_03
31 tháng 1 2019 lúc 8:37

Từ \(\frac{1}{1+x}+\frac{1}{1+y}+\frac{1}{1+z}\ge2\)

\(\Rightarrow\frac{1}{1+x}\ge\left(1-\frac{1}{1+y}\right)+\left(1-\frac{1}{1+z}\right)\)          

                    \(=\frac{y}{1+y}+\frac{z}{1+z}\ge2\sqrt{\frac{yz}{\left(1+y\right)\left(1+z\right)}}\)

C/m tương tự cũng có \(\frac{1}{1+y}\ge2\sqrt{\frac{xz}{\left(1+x\right)\left(1+z\right)}}\)

                                    \(\frac{1}{1+z}\ge2\sqrt{\frac{xy}{\left(1+x\right)\left(1+y\right)}}\)

Nhân 3 vế của các bất đẳng thức trên lại ta được

\(\frac{1}{\left(1+x\right)\left(1+y\right)\left(1+z\right)}\ge\frac{8xyz}{\left(1+x\right)\left(1+y\right)\left(1+z\right)}\)

\(\Rightarrow1\ge8xyz\)

\(\Leftrightarrow xyz\le\frac{1}{8}\)

Dấu "='' khi \(x=y=z=\frac{1}{2}\)

Vậy .......

Bình luận (0)
Ngô Tiên Phong
31 tháng 1 2019 lúc 14:49

Đây là môn toán mà!

Bình luận (0)
liên hoàng
Xem chi tiết
liên hoàng
Xem chi tiết
Nguyễn Hưng Phát
Xem chi tiết
Dương Lam Hàng
6 tháng 7 2018 lúc 8:04

Ta có: \(\frac{1}{x+1}+\frac{1}{y+1}+\frac{1}{z+1}=2\) (Như đề là lớn hơn hoặc bằng 2)

\(\Leftrightarrow\frac{1}{x+1}=2-\frac{1}{y+1}-\frac{1}{z+1}\)

                    \(=\left(1-\frac{1}{y+1}\right)+\left(1-\frac{1}{z+1}\right)\)

                      \(=\frac{y}{y+1}+\frac{z}{z+1}\ge2\sqrt{\frac{yz}{\left(y+1\right)\left(z+1\right)}}\)  (Vì x;y;z là ba số dương nên Áp dụng BĐT Côsi)

\(\Leftrightarrow\frac{1}{x+1}\ge\frac{2\sqrt{yz}}{\sqrt{\left(y+1\right)\left(z+1\right)}}\left(1\right)\)

Chứng minh tương tự ta được: \(\frac{1}{y+1}\ge\frac{2\sqrt{xz}}{\sqrt{\left(x+1\right)\left(z+1\right)}}\) (2)

                                                \(\frac{1}{z+1}\ge\frac{2\sqrt{xy}}{\sqrt{\left(x+1\right)\left(y+1\right)}}\) (3)

Nhân (1);(2);(3) ta có: \(\frac{1}{x+1}.\frac{1}{y+1}.\frac{1}{z+1}\ge\frac{2\sqrt{yz}}{\sqrt{\left(y+1\right)\left(z+1\right)}}.\frac{2\sqrt{xz}}{\sqrt{\left(x+1\right)\left(z+1\right)}}.\frac{2\sqrt{xy}}{\sqrt{\left(x+1\right)\left(y+1\right)}}\)

\(\Leftrightarrow\frac{1}{\left(x+1\right)\left(y+1\right)\left(z+1\right)}\ge\frac{8\sqrt{\left(xyz\right)^2}}{\sqrt{\left[\left(x+1\right)\left(y+1\right)\left(z+1\right)\right]^2}}\)

Với x;y;z > 0 ta có: \(1\ge\frac{8xyz}{\left(x+1\right)\left(y+1\right)\left(z+1\right)}.\left(x+1\right)\left(y+1\right)\left(z+1\right)\)

                     \(\Leftrightarrow1\ge8xyz\Leftrightarrow xyz\le\frac{1}{8}\)

Dấu "=" xảy ra <=> \(\hept{\begin{cases}\frac{x}{x+1}=\frac{y}{y+1}\\\frac{y}{y+1}=\frac{z}{z+1}\\\frac{z}{z+1}=\frac{x}{x+1}\end{cases}}\Leftrightarrow\hept{\begin{cases}x=y\\y=z\\x=z\end{cases}\Leftrightarrow x=y=z}\)

Vậy GTLN của xyz = 1/8 khi và chỉ khi x=y=z

P/S: Bài giải của em còn nhiều sai sót, mong mọi người thông cảm, góp ý

Bình luận (0)
Easylove
Xem chi tiết
Nguyễn Việt Lâm
16 tháng 7 2020 lúc 22:37

\(\frac{1}{1+x}\ge1-\frac{1}{1+y}+1-\frac{1}{1+z}=\frac{y}{1+y}+\frac{z}{1+z}\ge2\sqrt{\frac{yz}{\left(1+y\right)\left(1+z\right)}}\)

Tương tự: \(\frac{1}{1+y}\ge2\sqrt{\frac{xz}{\left(1+x\right)\left(1+z\right)}}\) ; \(\frac{1}{1+z}\ge2\sqrt{\frac{xy}{\left(1+x\right)\left(1+y\right)}}\)

Nhân vế với vế:

\(\frac{1}{\left(1+x\right)\left(1+y\right)\left(1+z\right)}\ge\frac{8xyz}{\left(1+x\right)\left(1+y\right)\left(1+z\right)}\)

\(\Rightarrow xyz\le\frac{1}{8}< 8\) (đpcm)

Chắc bạn ghi sai đề bài :)

Bình luận (0)
tiểu an Phạm
Xem chi tiết
Đinh quang hiệp
11 tháng 5 2018 lúc 20:49

áp dụng bđt cosi ta có:

\(x^3+y^3+1>=3xy\Rightarrow\frac{1}{x^3+y^3+1}< =\frac{1}{3xy}\)

tương tự \(\frac{1}{y^3+z^3+1}< =\frac{1}{3yz};\frac{1}{z^3+x^3+1}< =\frac{1}{3zx}\)

dấu = xảy ra khi x=y=z=1(thỏa mãn vì khi đó xyz=1*1*1=1)

\(\Rightarrow A< =\frac{1}{3xy}+\frac{1}{3yz}+\frac{1}{3zx}\)

\(\Rightarrow\)max của A là \(\frac{1}{3xy}+\frac{1}{3yz}+\frac{1}{3zx}\)khi x=y=z=1

khi đó A=\(\frac{1}{3\cdot1\cdot1}+\frac{1}{3\cdot1\cdot1}+\frac{1}{3\cdot1\cdot1}=\frac{1}{3}+\frac{1}{3}+\frac{1}{3}=1\)

vậy max A là 1 khi x=y=z=1

Bình luận (0)
Đặng Thanh Quang
11 tháng 5 2018 lúc 21:58

Với x, y>o ta có bđt \(a^3+b^3\ge ab\left(a+b\right)\Rightarrow a^3+b^3+1\ge ab\left(a+b\right)+1=ab\left(a+b\right)+abc=ab\left(a+b+c\right)\)

\(\Rightarrow\frac{1}{a^3+b^3+1}\le\frac{1}{ab\left(a+b+c\right)}=\frac{c}{a+b+c}\)

Cmtt ta được A\(\le\frac{a+b+c}{a+b+c}=1\)

Dấu = xra khi a=b=c và abc=1 =>a=b=c=1

Bình luận (0)
Pham Quoc Cuong
11 tháng 5 2018 lúc 22:50

Chứng minh BĐT \(x^3+y^3\ge xy\left(x+y\right)\)x,y>0 

\(\Leftrightarrow x^3+y^3-x^2y-xy^2\ge0\Leftrightarrow\left(x-y\right)^2\left(x+y\right)\ge0\)(đúng) 

Khi đó \(x^3+y^3+1\ge xy\left(x+y\right)+1=xy\left(x+y\right)+xyz=xy\left(x+y+z\right)\) 

\(\Rightarrow\frac{1}{x^3+y^3+1}\le\frac{1}{xy\left(x+y+z\right)}=\frac{z}{xyz\left(x+y+z\right)}=\frac{z}{x+y+z}\) 

Tương tự với các hạng tử khác, Ta có:

\(A\le\frac{z}{x+y+z}+\frac{x}{x+y+z}+\frac{y}{x+y+z}=1\) 

Vậy MaxA = 1

Bình luận (0)